User Avatar
finnis12
Joined
Apr 2025
Subscription
Free
PrepTests ·
PT102.S2.Q25
User Avatar
finnis12
Tuesday, Sep 24 2024

Can we consider Jack's aunt to be someone who is not benefitted by Jack's decision?

0
PrepTests ·
PT106.S1.Q17
User Avatar
finnis12
Saturday, Sep 21 2024

I'm confused. If money that would have gone to the local economy is now being used in the mall, how would the local economy still slightly increase? Wouldn't this being drawing money away from the local economy?

0
PrepTests ·
PT116.S3.Q13
User Avatar
finnis12
Friday, Sep 20 2024

Here's why A is correct: 2b years ago, the sun was dimmer than now. If the sun was that dim right now, all the water would be frozen, but there is evidence of non-frozen water 3.8b years ago (when the sun was dim). So we need to figure out what is different about the situation back then vs. now that made liquids not freeze.

A says that the atmosphere holds less heat now than it did 3.8b years ago. Ok, great. There's a difference between 3.8b years ago and now. This would also explain why the liquid was not frozen since the heat in the atmosphere would have presumably kept the water from freezing. Since currently the atmosphere no longer holds that much heat, water would freeze if the sun was dimmer.

I hope that helped!

0
PrepTests ·
PT105.S1.Q21
User Avatar
finnis12
Thursday, Sep 19 2024

Got this wrong in BR bc I got thrown off by "both doctrines are wrong" and thought we needed an NA that proves both wrong. That is not what we need though. I think I went wrong by thinking of this as an MBT and trying to prove that both are wrong when an NA just needs to be something that the author agrees with.

Look at this as an AND OR relationship. To prove the conclusion that both doctrines are wrong, you need to prove the Economic doctrine is false AND the Psychological doctrine is false. But you can prove the Economic OR the Psychological doctrine are not exclusive to weaken the conclusion since the conclusion relies on both.

If we negate A - "It is not true that the first doctrine precludes any noneconomic factors in explanations of historical events", then Doctrine 1 could include that any factors, including psychological ones, are needed to explain historical events... Well the conclusion says that this Doctrine is incorrect bc there are events due to both economic and psychological forces but with negated A, Doctrine 1 can include both economic and psychological forces so it would not be true that both Doctrines are mistaken.

I hope this made sense. It definitely helps to try to write out the explanation of why this is correct.

0
PrepTests ·
PT144.S4.Q18
User Avatar
finnis12
Wednesday, Sep 18 2024

But isn't it true that while yes it's easier to transport the dogs by boat, it is not impossible to transport them via land? I'm viewing E as correct because the argument says that traveling by land was more difficult than traveling by boat and E agrees with that. Negating E would directly contradict what the author said. Is that the real reason why E is correct, is because it proves that they must have been transported by boat seems to make a huge assumption.

0
PrepTests ·
PT116.S2.Q18
User Avatar
finnis12
Saturday, Aug 17 2024

Can we say D is correct bc it is the sufficient assumption needed to make the conclusion, "health edu is prop" true? attempts to influence behavior -> health edu is prop. And in this case we are looking for the sufficient assumption. B and C are wrong bc they don't lead to the correct conclusion?

0
User Avatar
finnis12
Sunday, Aug 11 2024

I don't understand how you can't rule out C for the same reason we ruled out D

2
PrepTests ·
PT107.S3.Q19
User Avatar
finnis12
Friday, Jul 26 2024

Could someone explain why E is incorrect?

0
User Avatar
finnis12
Tuesday, Jul 23 2024

Me too. Is this bad? I haven't used any lawgic on these but have gotten them all right (although I have not submitted a single one with confidence) Not sure if worth learning the lawgic?

2
PrepTests ·
PT128.S2.Q17
User Avatar
finnis12
Sunday, Jul 21 2024

I interpreted that the archeologists concluded that discoloration depends on time/season of death. If neanderthals did migrate and the gazelle teeth were found bc they saved them for ritualistic purposes, then wouldn't the teeth have no variation in discoloration that indicates they were killed in different seasons?

0
User Avatar
finnis12
Wednesday, Jun 26 2024

I don't get how acting like you're confident causes becoming genuinely confident.

0
User Avatar
finnis12
Wednesday, Jun 26 2024

Can someone help explain this one. I really do not get it.

2
User Avatar
finnis12
Friday, Jun 07 2024

This is invalid. (it would be valid if they were "most" statements)

Think about the bucket example. There are three buckets - A, B, C. Put some scoops of A in the B bucket and at the same time put some scoops of A in the C bucket. The B and C buckets never mixed. It would be invalid to say that some scoops of the B bucket are in the C bucket and vice versa. They just both have some scoops of the A bucket.

1
User Avatar
finnis12
Friday, Jun 07 2024

For me, an easy way to do this is by checking if the conclusion, in this case, "Therefore, the monarch butterfly does not migrate south in winter (x/B)," matches the sufficient. So in lawgic I would write

All Birds -> migrate south for winter A -> B

Monarch/Bird x /A

*now check, is the "/A" in the sufficient

spot? No. Ok so try the contrapositive

/B -> /A

*now is the "/A" in the sufficient? Still no.

So it is not valid that x/A -> x/B

I hope this helps sorry if it was confusing

1
User Avatar
finnis12
Thursday, Jun 06 2024

I find it easier to keep chains broken apart like this.

decline -> corrupt

corrupt -> revolution

q/corrupt

_

q/decline

Because this way it's easier to see what result "q/corrupt" gives us since we can see the sufficient assumption and necessary assumption in each line. We have to remember that "q/corrupt" only applies to the sufficient assumption. Keeping it broken apart, look at the first line, "decline -> corrupt", take the contrapositive, "/corrupt -> /decline" and since "q/corrupt" matches the sufficient assumption then we know that "q/declines".

Now looking at the second line, "corrupt -> revolution", if we take the contrapositive, "/revolution -> /corrupt" we see that q/corrupt matches the necessary condition, but we can ignore it because it has to match the sufficient assumption to reach a conclusion.

Sorry this was confusing and drawn out but basically I think it's easier to keep the chain broken apart so it's easier to see whether the "q/corrupt" matches the sufficient assumption from which we can infer the necessary assumption to apply,

0
User Avatar
finnis12
Wednesday, Jun 05 2024

This was incredibly helpful thank you!

0
User Avatar
finnis12
Wednesday, Jun 05 2024

For the first example, you want to negate the first clause (/first clause -> second clause) which would change "cannot" to "can" and use the if, then format. "If you can ride the roller coaster if you are under 4 feet tall, then you are older than 16 years old." The contrapositive would negate the second clause and put it first (/second clause -> first clause), " If you are not older than 16 years old, then you cannot ride the roller coaster if you are under 4 feet tall".

0
User Avatar
finnis12
Monday, Jun 03 2024

The necessary condition always falls on the right of the indicator arrow.

2
User Avatar
finnis12
Monday, Jun 03 2024

You are correct. How I think of it is by changing the wording to be a typical sufficient, necessary phrase (if, then). In this case it would read, "If the knight can kill a dragon (sufficient), then the knight wields an enchanted sword (necessary)." If the knight can kill a dragon, it must be true that the knight has an enchanted sword and if the knight does not have an enchanted sword, then they cannot kill the dragon. However, the knight could have an enchanted sword and just not kill the dragon. But if he did kill the dragon then he must have an enchanted sword.

0
User Avatar
finnis12
Saturday, Jun 01 2024

#help. On 7.2 and 7.3 why is "humidity" and "water" not included in their respective sentences?

0
User Avatar
finnis12
Friday, May 31 2024

I'm having trouble identifying the predicate-object. Does anyone have any tips for this?

0
User Avatar
finnis12
Friday, May 31 2024

#help In the 4th sentence, why is the predicate-object "the views" and not "scientists"

1
User Avatar
finnis12
Thursday, May 30 2024

While this is a sub sub conclusion, it does also support the sub conclusion/major premise that “Maintenance of medicine stock even at exorbitant expense remains indisputably worthy.” (remember that a sub conclusion, or in this case sub sub conclusion, is a claim that receives and gives support) So in this case, the sub sub conclusion receives support from the minor minor premise, "Single source of vital medicine rests only in one hospital within this devastated war zone," (which makes it a conclusion since it's receiving support), and it also gives support to the sub conclusion/major premise, "Maintenance of medicine stock even at exorbitant expense remains indisputably worthy.”

I hope this helped!

1
User Avatar
finnis12
Thursday, May 30 2024

The minor-minor premise supports the sub-sub conclusion. The sub-sub conclusion + minor-minor premise supports the sub conclusion/major premise. The sub conclusion/major premise + major premise supports the main conclusion.

0
User Avatar
finnis12
Thursday, May 30 2024

Adding onto what cegraystone wrote, the major premise just supports the conclusion better than the minor premise. If the major premise weren't included, then the minor premise would still support the conclusion it just wouldn't be as strong of an argument.

1

Confirm action

Are you sure?